LSAT and Law School Admissions Forum

Get expert LSAT preparation and law school admissions advice from PowerScore Test Preparation.

 Administrator
PowerScore Staff
  • PowerScore Staff
  • Posts: 8916
  • Joined: Feb 02, 2011
|
#36649
Complete Question Explanation

Strengthen. The correct answer choice is (B)

The chemist quoted here discusses a weed killer that is always present in two forms that are mirror
images of each other. One of the forms of this weed killer is deadly to weeds, as one might expect,
while the other has no effect. Local soil conditions are often more conducive to the breakdown
of one of these molecules over the other, and the variation in relative concentrations can strongly
influence the weed-killer’s effectiveness.

Based on these facts, the author concludes that a lot of the data on the weed-killer’s effects must be
misleading….what does this argument tell us about that data? If the author believes this data to be
misleading (based on the variation in concentration that depends on the particular soil sample) then
the author must believe that the data fails to account for the variation in concentration of the two
molecules.

The question that follows asks for the answer choice that most strengthens the author’s argument
—the choice that bolsters the conclusion that the weed-killer data is likely to be misleading (and
perhaps doesn’t consider the variation in molecular concentration).

Answer choice (A): The stimulus provided that the particular weed killer that’s being discussed is
present in two forms, one of which is harmless to weeds, and the other of which kills them. This
choice provides that this is generally the case with two-molecule weed killers. Even if this is the
general trend, this does not strengthen the author’s case that a lot of the data on the weed-killer is
probably misleading.

Answer choice (B): This is the correct answer choice—and the choice that strengthens the case
that the data on the weed-killer is likely misleading. If the vast majority of data on the product is
based on studies of equal concentration of the two molecules, then this wouldn’t account for the
concentration changes that come from variation in local soil conditions. This certainly helps the
author’s case that much of the data on the weed-killer is probably misleading.

Answer choice (C): Even if most local soil conditions have a higher concentration of the killermolecule,
this provides no information about the data referenced in the author’s conclusion, so
this answer choice does not strengthen the author’s conclusion that much of this data is probably
misleading.

Answer choice (D): This answer choice would weaken the author’s argument: if the data mentioned
by the author does a good job approximating the normal conditions in which the weed-killer is
generally used, this would damage the author’s argument that such data is likely to be misleading.

Answer choice (E): This one might be initially appealing, since it deals with weak data—a biased
study based in which half the information is missing! The problem is that this answer only provides
that such data would almost certainly be misleading—the answer does not provide that the data on
this weed-killer has such omissions. Since this choice does not help the author’s case that the data on
this specific weed-killer is likely to be misleading, it cannot be the correct answer to this Strengthen
question.
 Winup12
  • Posts: 8
  • Joined: Dec 02, 2011
|
#3423
The conclusion of the stimulus is that "the data...misleading".

The correct choice B states that the data are drawn from equally controlled conditions, which should result in comparable results. Does that mean the lab test is too well controlled and different from the real world conditions? And the "misleading" is referring to the lab test data? I am confused.

Any explanation is appreciated. Thanks!
 Steve Stein
PowerScore Staff
  • PowerScore Staff
  • Posts: 1153
  • Joined: Apr 11, 2011
|
#3442
Thanks for your question. In that one, as you correctly point out, the author's conclusion is that the data are probably misleading. Let's take a look at how the argument breaks down:

Premise: The relative concentration of the two molecules will vary significantly based on the soil.

Conclusion: Much of the data are probably misleading.

So, the author believes that the data from the lab won't reflect what actually happens "in the field." To strengthen this argument, we will try to bolster the claim that the lab data won't match what actually happens in practice.

Answer choice B does this: The more of the lab data that was done with a perfect 50/50 concentration of the molecules, the more likely that in practice, variation in concentration will yeild different results.

That one is tricky--let me know whether this clears it up--thanks!


~Steve
 eober
  • Posts: 107
  • Joined: Jul 24, 2014
|
#16675
Hi,

I am very confused how answer choice B strengthens the argument. In the argument we are told that there are two forms of the molecule - one has effect, the other has no effect on killing weeds. Their level of concentration depends on the soil since it determines which one of the molecule forms will be broken down. Chemist concludes that studies are not reliable in showing the effect of weed-killer. Shouldn't there be something wrong with the experimentation for the data to be misleading? How does having equal concentrations of the molecule be helpful to the argument?

Also, what is the prephrase you formed in this question?

Thanks! :)
 Robert Carroll
PowerScore Staff
  • PowerScore Staff
  • Posts: 1787
  • Joined: Dec 06, 2013
|
#16723
eober,

The chemist claims that local soil conditions will usually favor the breakdown of one form or the other. That is, local soil conditions tend to be biased toward one form or another, so that the forms are not evenly distributed in real-world conditions.

Answer choice (B), adding, as it can for a Strengthen question, new information, gives us a reason to doubt the quality of the data - the laboratory conditions were situations where the two forms were equally likely to break down. As most real-world situations are situations where there is a bias toward one form or another, a laboratory experiment where there is no such bias, where the two forms are equal, is per se different from most real-world situations. This gives us a reason for doubting those laboratory results, which supports the chemist's skepticism about them.

As far as a prephrase, in this stimulus, the chemist's conclusion talks about the data's being misleading, but the rest of the stimulus said nothing about the data, so this is a situation with new information in the conclusion. I need to find a connection between the conclusion (data are misleading) and what the premises talk about. So my prephrase will have to involve something about the data's not being collected in a situation that effectively tests the weed-killer.

Robert Carroll
 LustingFor!L
  • Posts: 80
  • Joined: Aug 27, 2016
|
#35593
Can someone walk me through why D is incorrect? I was between B and D. Looking back, I can se that B strengthens the data more, because it brings up the concentrations mentioned in one of the premises.
 Adam Tyson
PowerScore Staff
  • PowerScore Staff
  • Posts: 5153
  • Joined: Apr 14, 2011
|
#36069
To start with, Lusting, you've already answered your own question. Why is D incorrect? Because B is better! We were asked to pick the one that strengthens more, and the instructions say to pick the best answer. There's no need to dig deeper than that, and any attempt to do so during the test wastes time and effort better spent elsewhere.

However, we can learn through analysis after a practice test or series of practice questions why one answer is better than another, so let's delve into this and see what we can learn.

The conclusion that we want to strengthen is that the test results are probably misleading. Our prephrase should therefore be that something makes the results misleading - a problem in the study, most likely. Does answer D give us a problem in the study that would make the results misleading? Not at all! Just the opposite, in fact - if we studied the weed killer in a variety of conditions, and we simulated real-world conditions, that would make the study better and the results more reliable and accurate, not more misleading. D is, upon closer look, an opposite answer, weakening the claim that the results are misleading rather than strengthening is as we were asked to do.

It's easy to get turned around on a strengthen question and start to look for weaken answers, and vice versa, especially when you are asked to strengthen that something is bad or weaken something that's good. Don't lose sight of what you are trying to do to the argument and what that argument said, and you will be less likely to get turned around in these ways.

Continued good luck to you, Lusting!
User avatar
 lsatlearner105
  • Posts: 2
  • Joined: Aug 11, 2021
|
#89536
I understand that (B) is the correct answer because the "laboratory" studies (non-bias) do not reflect real-world soil conditions (bias to one of the two molecules) meaning that it would support the author's conclusion that the data is misleading; however, I'm unsure why (E) is incorrect.

The explanation by Adam says:
"Answer choice (E): This one might be initially appealing, since it deals with weak data—a biased
study based in which half the information is missing! The problem is that this answer only provides
that such data would almost certainly be misleading
the answer does not provide that the data on
this weed-killer has such omissions
. Since this choice does not help the author’s case that the data on
this specific weed-killer is likely to be misleading, it cannot be the correct answer to this Strengthen
question."

To be 100% clear, is this answer choice wrong because it fails to accurately mention how misleading it is (I.e. almost certainly be misleading); therefore, the lack of complete certainty doesn't provide the most support? Alternatively, is (E) incorrect because it also doesn't mention the data relevant to the "specific" (stimulus mentioned "certain") weed killer meaning that this answer choice is speaking about weed killers broadly (which is out of scope of what is needed for the author's conclusion)?


Any help with this would be very great! Thank you!
User avatar
 atierney
PowerScore Staff
  • PowerScore Staff
  • Posts: 215
  • Joined: Jul 06, 2021
|
#89565
Hi,

Generally speaking, remember on strengthen questions, we're looking for the best answer. What this means is that, while I appreciate your questions on why a certain choice is "wrong," and will certainly endeavor to answer them to your satisfaction, remember that essentially every answer choice on a strengthen is wrong primarily because it does not strengthen the argument to the degree that the correct answer choice. Thus, my approach on these questions has always been to compare the relative strength of the answer choices in terms of their ability to make the argument more plausible. For the reasons provided above, B certainly does that. And really, the question you want to ask is why B strengthens more so than E. And I believe this has adequately explained above, so I won't address it here.

In terms of E though, by itself, E is definitely a statement an intermediate conclusion of sorts, by relying upon the idea that the two forms of weed-killer deliver vastly distinct results, it concludes that data only examining one of the two forms will almost certainly be misleading. This is a valid intermediate conclusion, regardless of the specificity (or relative absence thereof) of how misleading the data is claimed to be. Thus, in answer to your first question, I would say that no, the fact that claims data would be certainly misleading is not, by itself, the be the reason that E is the best answer here. However, the intermediate conclusion really has no teeth without the specific data used; hence the answer to your second question would be "yes," answer choice E would need some type of indication that the data for this particular weed-killer relied solely on examination of the effects of one of its two molecular forms.

Notice that this is exactly what B provides, specific evidence to bridge inherent in this strengthen (must like you would see those of justify the conclusion or sufficient assumption inquiries).

Let me know if you have further questions on this.

Get the most out of your LSAT Prep Plus subscription.

Analyze and track your performance with our Testing and Analytics Package.